1answer.
Ask question
Login Signup
Ask question
All categories
  • English
  • Mathematics
  • Social Studies
  • Business
  • History
  • Health
  • Geography
  • Biology
  • Physics
  • Chemistry
  • Computers and Technology
  • Arts
  • World Languages
  • Spanish
  • French
  • German
  • Advanced Placement (AP)
  • SAT
  • Medicine
  • Law
  • Engineering
Sveta_85 [38]
3 years ago
7

If f(x)=e^2x(x^3+1) then f'(2)=

Mathematics
1 answer:
qaws [65]3 years ago
7 0

Answer:

See below

Step-by-step explanation:

f(x)=e^2x(x^3+1) = e^2x^4+x

Considering

f'(x)=\dfrac{d}{dx}f(x)=\dfrac{d}{dx}e^2x^4+\dfrac{d}{dx}x

Once the derivative of a constant is 1 and  e^x = \dfrac{d}{dx} e^x

Then,

\dfrac{d}{dx}f(x)=\dfrac{d}{dx}e^2x^4+\dfrac{d}{dx}x =\boxed{ e^2 4x^3+1}

Therefore,

f'(2) = e^24\cdot 2^3 +1 = 32e^2+1

You might be interested in
WHICH IS A TRUE STATEMENT BASED ON THE DOT PLOTS BELOW 10 POINTS
Nata [24]

Answer:

C. Set B has the greater range.

Step-by-step explanation:

We can start out with listing out each set of data in a way that makes it easier to work with.

Set A:

20,20,30,30,40,40,40,40,40,50,50,60,60,60

Set B:

10,10,20,20,20,30,30,30,30,40,50,50,60

We can start by trying to eliminate answer options; starting with D.

To get the mean of a set of data, we need to add all numbers in the data set and then divide by the number of dots on the plot.

For Set B: 10+10+20+20+20+20+30+30+30+30+40+50+50+60=420 then divide by number of dots (14) 420/14=30 The mean of Set B is equal to 30.

Solve the mean for Set A to compare. The mean is 41. D is not true so we can eliminate this answer choice.

We can look at the median of both sets so that we can eliminate answer choice B. The median is the middle most point in the set of data.

Set A median: 40

Set B median: 30

We can eliminate B because it does not have the greater median.

On to option C: range.

To find the range of each data set, we subtract the smallest number from the biggest number.

Set A: 60-20=40

Set B: 60-10= 50

Set B does in fact have a greater range than Set A. Answer choice C is the correct option.

6 0
3 years ago
The movie theatre has 250 seats. 225 seats were sold for the current showing . What percent of the seats are empty?
Leno4ka [110]
250-225=25 seats empty. Place what you want/total=(25/250)x100=0.1x100=10% of the seats are empty.
6 0
3 years ago
Read 2 more answers
If the area of the right triangle is 72 what is the value of x
wel

36 i guess ha please that not right

8 0
3 years ago
Ali has x cards.
valentina_108 [34]

━━━━━━━━━━━━━━━ ♡ ━━━━━━━━━━━━━━━  

If Ali has x cards, then Belinda has 2x cards.

If Ali has x cards, then Charlie has x+5 cards.

2x (Belinda) + x+5 (Charlie) + x (Ali) = 33 (total)

2x + x+5 + x = 33

Now simplify the equation... x+x+5 can be simplified as 2x+5.

2x+5 plus 2x can become 4x+5.

So their total number is 4x+5.

So 4x+5=33.

You want to isolate the variables and numbers.

To remove the 5 from the left side, subtract 5 from both sides.

4x+5-5=4x - 33-5=28

So the equation now is 4x=28

To remove the 4, divide each side by 4.

4x÷4=x - 28÷4=7

So x equals 7, Ali has 7 cards.

━━━━━━━━━━━━━━━ ♡ ━━━━━━━━━━━━━━━  

5 0
3 years ago
Read 2 more answers
If 5 inches represent 15 feet, how many inches would represent 45 feet?
r-ruslan [8.4K]

Answer:

26 rows · An inch is a unit of length equal to exactly 2.54 centimeters. There are 12 inches in a foot, and 36 inches in a yard. There are 12 inches in a foot, and 36 inches in a yard. Feet to Inches Conversions

Step-by-step explanation:

5 0
2 years ago
Other questions:
  • Which polygon will always be irregular?
    8·2 answers
  • 20 POINTS<br> HELP DUE IN AN HOUR
    10·1 answer
  • Find the measure of b
    11·1 answer
  • Simplify 7y+7-8(y-7)
    5·2 answers
  • Julio ​says, "If you subtract 16 from my number and multiply the difference by −6​, the result is −138​." What is Julio​'s ​numb
    7·1 answer
  • Can someone help me please​
    7·1 answer
  • This package of cheese costs $2.97. How much would a package with 18 slices cost at the same price per slice?
    11·1 answer
  • Plz hurry
    7·1 answer
  • What is $82.51 minus $1?
    12·2 answers
  • Need B and C PLEASE HELP
    6·1 answer
Add answer
Login
Not registered? Fast signup
Signup
Login Signup
Ask question!